2008 AMC 12A Problems/Problem 1

Revision as of 14:28, 17 February 2008 by Xantos C. Guin (talk | contribs) (New page: ==Problem == What is the reciprocal of <math>\frac{1}{2}+\frac{2}{3}</math>? <math>\textbf{(A)} \frac{6}{7} \qquad \textbf{(B)} \frac{7}{6} \qquad \textbf{(C)} \frac{5}{3} \qquad \textb...)
(diff) ← Older revision | Latest revision (diff) | Newer revision → (diff)

Problem

What is the reciprocal of $\frac{1}{2}+\frac{2}{3}$?

$\textbf{(A)} \frac{6}{7} \qquad \textbf{(B)} \frac{7}{6}  \qquad \textbf{(C)} \frac{5}{3}  \qquad \textbf{(D)}  3  \qquad \textbf{(E)}  \frac{7}{2}$

Solution

$\left(\frac{1}{2}+\frac{2}{3}\right)^{-1}=\left(\frac{3}{6}+\frac{4}{6}\right)^{-1}=\left(\frac{7}{6}\right)^{-1}=\frac{6}{7}\Rightarrow A$.

See Also